Math, asked by sarveshjha, 4 hours ago

a 4,128+______=4,128
b _______+82+ 71=_______+96+82




Answers

Answered by bigdashezada61
7

Correct option is

A

36,0.44

⇒ The given numbers are 59,46,30,23,27,40,52,35,29.

⇒ Here, the largest value =x

m

=59 and the smallest value =x

0

=23.

⇒ Range=x

m

−x

0

=59−23

=36

⇒ Coefficientofrange=

x

m

+x

0

x

m

−x

0

=

59+23

59−23

=

82

36

=0.44

Answered by aWitSu
0

Answer:

a. 0

b true

Step-by-step explanation:

Similar questions